LSAT and Law School Admissions Forum

Get expert LSAT preparation and law school admissions advice from PowerScore Test Preparation.

 Daniel B
  • Posts: 5
  • Joined: Sep 08, 2017
|
#39968
Based on the explanation and all of the diagrams, how would it be possible to answer this question in 1 min and 20 seconds?
 AthenaDalton
PowerScore Staff
  • PowerScore Staff
  • Posts: 296
  • Joined: May 02, 2017
|
#39978
Hi Daniel,

Thanks for your question!

Some questions on the LSAT are more difficult than others, so some will take more than 1 minute, 20 seconds and others will take less. This question requires a bit more time.

However, there are some ways to speed up the process of finding the right answer. First, you don't have to diagram every inference to tackle this question (although we did so here for the sake of a thorough explanation). The diagrams you should use in your practice tests should use abbreviations, not full phrases.

Second, you may be able to eliminate a few answer choices without working through all the diagrams, which will narrow your options. In this question we are told that negotiations will take place soon. The third sentence of the stimulus discusses a variety of circumstances that must be in place if negotiations take place: international troops being able to enforce a cease-fire, a major incentive to resume fighting has been suppressed, etc. So just based on that sentence, we can easily conclude that answer choices (B) and (C) must be true and are therefore incorrect answers. We also know from the second sentence that if negotiations are held, other countries have exerted pressure -- which allows us to easily eliminate answer choice (D).

By quickly narrowing the answer choices down to (A) and (E), you can focus your time on the inferences that support just two answer choices, instead of all five.

Finally, this will all become much faster with practice. :)

Best of luck studying!

Athena
 elewis10
  • Posts: 21
  • Joined: Sep 02, 2017
|
#44843
i don't see how A is a mistaken reversal. it only gives part of the sufficient --> necessary. i'm having trouble understanding why C is wrong. can someone explain to me? thank you!
 Adam Tyson
PowerScore Staff
  • PowerScore Staff
  • Posts: 5191
  • Joined: Apr 14, 2011
|
#44946
Take a look back at the first post in this thread, elewis10, which is the administrator's official explanation, and you will see that we can diagram part of the stimulus as showing that if nobody violated the cease fire, negotiations must have begun (or will soon). Answer choice A is a Mistaken Reversal of that claim, because negotiations beginning is not sufficient to prove that nobody has violated the cease fire, but rather is necessary in that case. While it may be true that nobody has violated the cease fire, it could also be true that someone has violated it and negotiations began anyway!

As to answer choice C, if negotiations will begin soon, per the question stem, then we know that troops have demonstrated their abilities (per the last sentence of the stimulus), and we know further that a major incentive has been suppressed (also per that last sentence). Since we can follow the conditional chain from "negotiations begun" all the way to "major incentive has been suppressed", answer C is something that must be true, and is therefore an incorrect answer in this EXCEPT question.

Check that first post again, build your conditional chain, and follow it to all of its logical conclusions, being careful not to go backwards along the way. The only way to change directions in a conditional argument is to start by negating something, and then all the arrows flip the other way for you.
 abutz
  • Posts: 9
  • Joined: Apr 11, 2020
|
#75575
I have been trying for about 15 minutes to understand this, and for some reason I cannot. It's making me feel ridiculously stupid!

It seems to me that the cease fire will not be violated if the negotiations begin soon. Wouldn't the contrapositive of that be (negate)Negotiations begin soon → (negate)Cease fire Will be violated/cease fire will not be violated? So wouldn't answer choice A have to happen? If the negotiations between the two sides do begin, then doesn't it mean that the cease fire would not have been violated by the two sides? I'm so confused. I've read all of the comments and it is just confusing me more for some reason.

It is said to be mistaken because when it gives us "if negotiations between the two sides do begin soon," in our diagramming we don't have any part that has negotiations begin soon ---> __________
We only have (negate) violated & (negate) negotiations begin soon as choices on the left hand side of diagrams to "push the buttons in order to make the necessary happen. Am I thinking along the right terms here? Is this why A is the answer?
 Frank Peter
PowerScore Staff
  • PowerScore Staff
  • Posts: 99
  • Joined: May 14, 2020
|
#75602
Hi Abutz,

The first statement in the stimulus can be diagrammed as ~NBS --> CFV. The contrapositive would be ~CFV --> NBS.

In the question stem, the four incorrect answer choices will be statements that can be inferred as true from the fact that negotiations have begun. For the purposes of addressing answer choice (A) let's just focus on the above conditional statement. The state of affairs where negotiations have begun (NBS) is on the necessary side in our contrapositive statement. When we're simply told that we have a necessary condition (as we're told in the question stem), there's really nothing we can infer, which is why (A) is the correct answer here.
It seems to me that the cease fire will not be violated if the negotiations begin soon. Wouldn't the contrapositive of that be (negate)Negotiations begin soon → (negate)Cease fire Will be violated/cease fire will not be violated?
Your first statement is correct (~CFV --> NBS), but the contrapositive would be ~NBS --> CFV.
User avatar
 SGD2021
  • Posts: 72
  • Joined: Nov 01, 2021
|
#92393
Hello,

In this question, it is essentially as though we are given the sufficient condition in the question stem and then we have to check if the answer choice correctly follows from that sufficient condition, such as by checking if the answer choice is the correct necessary condition associated with that sufficient condition given in the question stem, correct? Does this kind of thing appear often on the LSAT?

Also, when solving a diagramming question, if we can use our diagram to find the exact answer among the answer choices, should we still read all answer choices or move on as soon as we find a match? For a question like this, for example, should we read all answer choices if we already found A to be correct based on our diagrams?
 Rachael Wilkenfeld
PowerScore Staff
  • PowerScore Staff
  • Posts: 1392
  • Joined: Dec 15, 2011
|
#92406
I think that's a good analysis, SGD. This is a case I would certainly diagram out on paper, and I think the answers are fairly easy to see once you draw it out. The bulk of your time here will be spent on accurate diagramming.

The balance between speed and accuracy when moving on before finishing the answer choices is the main issue. For me, that small bit of time it takes to read through all five, especially in a case like this where I could quickly and easily check my diagram, is worth it for the accuracy bump. Even after well over a decade of doing this work, I catch errors by reading through all five choices.

Here's the way I think about it. In a typical LR section, I'll maybe have 2-3 questions where I'm fairly confident of the answer choice as soon as I see it. It won't save me enough time to do a whole extra question by skipping those last few answer choices. On the other hand, it could help me gain a point if it shows me an error I made.

Your calculations may be different. By test day, you might find that your score is higher when you move on without reading all five in a situation like this one. You might not. That's what your PTs are all about---discovering the ideal strategy for you.

Hope that helps!
User avatar
 Bmill05
  • Posts: 4
  • Joined: Dec 23, 2021
|
#92830
NEVERMIND, Still posting.

I believe I answered my own questions, so nevermind. I am still posting this response thread because typing it out helped make everything 'click' for me so maybe it will for someone else.

Dave,

I originally choice E for my answer to this question. While your explanation as to why A is the correct answer is the clearest to me in this thread, I am still struggling with why E must be true. Hopefully I can explain what I am struggling with clearly...

I now understand why A is an exception to what must be true. During the drill I had not realized that the answer choice swapped the sufficient and necessary because I incorrectly attempted to chain diagram the information in the stimulus. However, I do not understand why E is also not an exception.

E is given in the stimulus, however, the question stem is asking about what must be true at the start of negotiations, however, an agreement would come at the end?

I went to copy & paste part of the problem when it clicked. E is not talking about the end of negotiations. While the question stem mentions that "at the time those negotiations begin" the stimulus states that "an agreement will emerge only if other countries continue such pressure throughout the negotiations" thus, (E) must be true since pressure must be maintained throughout negotiations, which includes the beginning of negotiations.
 Robert Carroll
PowerScore Staff
  • PowerScore Staff
  • Posts: 1787
  • Joined: Dec 06, 2013
|
#92868
Bmill05,

Sounds like you got it! Let us know if you have any other questions.

Robert Carroll

Get the most out of your LSAT Prep Plus subscription.

Analyze and track your performance with our Testing and Analytics Package.